Hi glasann!
Even though this question only gives you a partial list of the order, I would still approach it like a traditional List question: use one rule at a time to eliminate answer choices.
Start with the most fixed rule, the first one: P

M

L.
If P is before M, M cannot be 1st, so answer choice (A) is eliminated.
If M is between P and L, we cannot have P 4th and L 5th, so answer choice (D) is eliminated.
Now onto the next rule: G is before both L and J or G is after both L and J. Basically, this rule means that G cannot be
between L and J.
If G is 5th and J is 6th, there is no room for L to also be after G, so answer choice (C) is eliminated.
If G is 5th and L is 6th, there is no room for J to also be after G, so answer choice (E) is eliminated.
The only answer choice we're left with is (B)--and we didn't even need to use the last rule or do any additional diagramming!
Hope this helps!
Best,
Kelsey